Was sind die eigentlichen Transformationseigenschaften von Dirac-Spinoren uσ(p)uσ(p)u_\sigma(p)?

Lassen u σ ( P ) ein Dirac-Spinor sein . Soweit ich weiß, wandelt es sich bei Änderungen des Bezugssystems entsprechend um

(1) u σ ( P ) = S ( Λ ) u σ ( Λ P )
bei dem die σ Etikett mischt sich nicht. Warum ist das? sollten die Polarisationen nicht frameabhängig sein? Schließlich ist die "Spin-Quantisierungsachse" frameabhängig.

Anders ausgedrückt: Die obige Beziehung ist äquivalent zu

(2) U ( Λ ) | P , σ = | Λ P , σ
ohne σ Mischen. Für mich sollten sich Ein-Teilchen-Zustände entsprechend umwandeln
(3) U ( Λ ) | P , σ = ? σ ' D σ σ ' ( Λ ) | Λ P , σ '
obwohl dies offensichtlich nicht der Fall ist. Wenn das wahr wäre, dann hätten wir es getan
(4) u σ ( P ) = ? S ( Λ ) σ ' D σ σ ' ( Λ ) u σ ' ( Λ P )
anstatt ( 1 ) .

Frage : warum mischen sich Polarisationen nicht unter Lorentz-Transformationen (in keiner ( 1 ) noch ( 2 ) )?


BEARBEITEN

Wie von Blazej hervorgehoben , die Spin-Komponenten σ Mischen Sie unter Lorentz-Transformation, und das richtige Gesetz ist

u σ ( P ) = σ ' M σ σ ' u σ ' ( Λ P )
für irgendeine Matrix M (was eigentlich mit einer kleinen Gruppenmatrix von Wigner verwandt ist, aber deren Form für mich nicht so relevant ist; der wichtige Teil ist, dass die σ Komponenten mischen, und nicht, was die eigentliche Matrix ist, die sie mischt).

Meine Sorge ist, dass dies nicht das ist, was ich online finde: Siehe zum Beispiel diese Antwort in Physics.SE (letzte Gleichung). Siehe auch diesen Wikipedia-Artikel . Wer hat Recht und wer hat Unrecht?

Beachten Sie, dass ( 3 ) ist auch keine korrekte Transformationsregel. Es sollte sein U ( Λ ) | P S = S ' D ( W ( Λ , P ) ) S ' S | Λ P , S ' , Wo W ( Λ , P ) ist Wigners Rotation. Wenden Sie sich für Einzelheiten an Weinberg, Band 1, Kapitel 2.
@Blazej Danke! Ich sehe nicht wirklich den Unterschied zwischen Ihrer Formel und meiner :-S Sie haben gerade die geschrieben D Matrix in allen Einzelheiten, aber der Inhalt unserer beiden Formeln ist gleich, richtig? (Ich hab geschrieben D σ σ ' ( Λ ) und du hast geschrieben D S S ' ( W ( Λ , P ) ) , aber das sind die gleichen, oder?).

Antworten (2)

Wir können diese Frage allgemeiner beantworten: Was sind die Transformationseigenschaften für die Polarisationen, die mit einem massiven Feld verbunden sind? (Ich beschränke mich auf massiv, weil die Polarisationen für masselose Teilchen mit Spin größer oder gleich 1 die Diskussion der Eichinvarianz erfordern; ich verschiebe das auf einen anderen Tag).

Die Polarisationen können ohne Bezugnahme auf die Feldgleichung definiert werden: Sie werden als Matrixelemente für ein Feld zwischen dem Vakuum- und dem Ein-Teilchen-Zustand definiert:

0 | ψ ( 0 ) | P , σ u σ ( P )
wobei der Proportionalitätsfaktor eine Konstante ist (bekannt als Wellenfunktionsrenormierung) in der relativistischen Normalisierung für den Zustand | P , σ . Hier der Index ist der Index in der vom Feld getragenen Lorentz-Darstellung ψ (Zum Beispiel, = μ für einen 4-Vektor, = a allgemeiner für einen Dirac-Spinor = ( a , β ) ist ein Paar von Indizes in der ( A , B ) Darstellung von S Ö ( 3 , 1 ) S U ( 2 ) × S U ( 2 ) ). Die Polarisationen tragen auch einen anderen Index, σ , der den Spin des Teilchens darstellt. Genauer gesagt ist es der Kleingruppenindex, den das Teilchen trägt, entweder der Spin oder die Helizität. Diese Definition sagt uns sofort wie u σ verwandelt sich angesichts dessen ψ trägt eine Lorentz-Darstellung
U ( Λ ) ψ ( X ) U 1 ( Λ ) = D ( Λ 1 ) ' ψ ' ( Λ X )
und die Ein-Teilchen-Zustandstransformation in Bezug auf die kleine Gruppe mit einer Wigner-Rotation
U ( Λ ) | P , σ = L σ ' σ ( W ( Λ ) , P ) | P Λ , σ '
was implizieren
D ( Λ ) ' u ' σ ( P ) = u σ ' ( P Λ ) L σ ' σ ( W ( Λ ) , P )
Wo P Λ ist 3-Vektor-Teil des 4-Vektors Λ P . (Man kann diese Gleichung auch lesen, indem man sagt, dass sich die Polarisationen links unter Lorentz und rechts unter den Kleingruppentransformationen transformieren: Das muss so sein, dass man die Lorentz-Indizes der Korrelationsfunktionen von Feldern in die umrechnen kann kleine Gruppenindizes der Streumatrix S wie von der LSZ-Reduktionsformel vorgegeben).

Dies beantwortet Ihre Frage. Aber eigentlich können wir noch mehr sagen: Diese Transformationseigenschaften sind konstruktiv, da sie es Ihnen erlauben, die Polarisation explizit zu bestimmen (und zu zeigen, dass sie bestimmte Gleichungen erfüllen, zB Dirac für Spin-1/2, ...), wie es vor langer Zeit gezeigt wurde in den 60er Jahren von Weinberg (siehe die Diskussion in seinem Lehrbuch über QFT, Band 1, Kapitel 5). Nehmen Sie zum Beispiel k = ( M , 0 ) (für ein massives Teilchen) und wenden Sie die kanonische Lorentz-Transformation an L = Λ das bringt es P = ( E , P ) = L k . In diesem Fall ist die Wigner-Rotation trivial, W = 1 , und daher

u σ ( P ) = D ' ( L ) u ' σ ( 0 )
was bedeutet, dass wir sie nur bei Nullimpuls kennen müssen (oder für masselose Teilchen in Bezug auf den Referenzvektor, der für die kleine Gruppe verwendet wird). Außerdem für eine beliebige Drehung Λ = R wir haben W = R für alle P so dass
D ' ( R ) u ' σ = u σ ' ( 0 ) L σ ' σ ( R )
Unter diagonalen Drehungen um die z Achse, L σ ' σ ( R z ) = e ich σ θ δ σ ' σ , kann die Polarisation durch das Infinitesimale extrahiert werden z -Drehung
D ' ( J z ) u ' σ 0 = σ u σ ( 0 ) .

Lassen Sie mich nur ein aufschlussreiches Beispiel geben: ein massiver Spin-1-Zustand (wo ist ein Index im irrep ( 1 / 2 , 1 / 2 ) , das ist = μ = 0 , 1 , 2 , 3 ist ein 4-Vektor-Index) hat eine dreidimensionale Darstellung von J z Wo D ( J z ) ich J = ich ϵ 3 ich J Und D ( J z ) 00 = D ( J z ) ich 0 = 0 damit die Polarisationen

ϵ μ ± ( 0 ) = 1 2 ( 0 , 1 , 1 , 0 ) T , ϵ μ 0 = ( 0 , 0 , 0 , 1 ) T .
Lösen Sie die gewünschten Gleichungen oben. Offensichtlich befriedigen sie P μ ϵ μ σ = 0 , sobald wir steigern ( M , 0 ) Zu P . Daher das Matrixelement
Ψ μ ( X ) 0 | ψ μ ( X ) | P , σ = e ich P X 0 | ψ μ ( 0 ) | P , σ e ich P X u μ σ ( P )
erfüllt
( + M 2 ) Ψ μ ( X ) = 0 , μ Ψ μ ( X ) = 0
die eher abgeleitet werden als Ausgangspunkt.

Dasselbe kann für jeden Spin gemacht werden, insbesondere für den Spin-1/2 und siehe, dass sie die Dirac-Gleichung lösen. Allgemeiner seit der Lorentz-Gruppe S Ö ( 3 , 1 ) S U ( 2 ) A × S U ( 2 ) B der Drehimpuls ist gegeben durch J = J A + J B was uns das sagt

D ( J A ) a a ' u a ' β σ ( 0 ) + D ( J B ) β β ' u a β ' σ ( 0 ) = L ( J S ) σ ' σ u a β σ ' ( 0 )
Wo = ( a , β ) . Mit anderen Worten, die Polarisationen sind (proportional zu) dem Clebsch-Gordan-Koeffizienten für den Spin S innen gefunden A B
u a β σ ( 0 ) C ( A B ) a β ( S ) σ
Einige der Eigenschaften der Polarisationen stammen tatsächlich aus der einheitlichen Bedingung für diese Clebsch-Gordan-Koeffizienten.

Also, um es einfach auszudrücken, die kurze Antwort ist, dass diese Links am Ende meines Posts einfach falsch sind? Jemand sollte den Wikipedia-Eintrag dann bearbeiten, oder?
@AccidentalFourierTransform Nein, diese Links sind korrekt, und das ist meine Antwort. Sie übersehen den Punkt, den ich sorgfältig zu betonen versucht habe: dass die Polarisationen zwei Indizes haben, ein Spinorial, das sich somit wie in diesen Links transformiert, und ein Kleingruppenindex, der sich ebenfalls transformiert, jedoch mit den Wigner-Rotationen. Es gibt keinen Widerspruch, diese Links diskutieren einfach den Fall von Objekten mit einem einzelnen Spinorialindex (wie zB einem Spin 1/2-Feld), während Polarisationen einen anderen, zusätzlichen, kleinen Gruppenindex haben. Sie können sogar sehen, dass meine zweite Gleichung mit dem übereinstimmt, was in den Links behauptet wird
@AccidentalFourierTransform Entschuldigung, ich habe nur auf die Wiki-Seite geschaut, die korrekt ist, wie ich in dem gerade geposteten Kommentar gesagt habe. Andererseits ist die alte Antwort auf SEhttp://physics.stackexchange.com/questions/87575/unitary-lorentz-transformation-on-quantized-dirac-spinor/228078#228078, die Sie verlinkt haben, einfach falsch.

Beachten Sie zunächst, dass u(ps)u(ps) keine Zustände im Hilbert-Raum einer Quantentheorie sind. Stattdessen sind sie Lösungen für bestimmte Gleichungen, nämlich ( γ μ P μ M ) u ( P S ) = 0 (äquivalent: u ( P S ) e ich P X löst die Dirac-Gleichung). Daher ist die Klammernotation nicht wirklich vorhanden (obwohl verlockend!). Die zweite Bemerkung ist, dass Sie darüber nachdenken sollten, wie der Spin überhaupt definiert ist? Die übliche Konvention besagt, dass der Spin eines sich bewegenden Teilchens als sein Spin in dem Referenzrahmen definiert ist, in dem es sich nicht bewegt. Nun lass P 0 = ( M , 0 , 0 , 0 ) und definieren u ( P 0 S ) als Lösung der Dirac-Gleichung, die ein ruhendes Teilchen mit Spin beschreibt S . Für jede Drehung (die nur Lorentz-Transformation ist Λ so dass Λ P 0 = P 0 ) haben wir eine Beziehung, die aus gewöhnlichem QM bekannt ist

S ( Λ ) u ( P 0 S ) = D ( Λ ) S ' S u ( P 0 S ' )
Dann für jeden möglichen Schwung P Von diesem Partikel wählen wir einen Standard-Boost Λ 0 ( P ) der sich verwandelt P 0 Zu P . Standardauswahl ist nur Boost in der P Richtung. Jetzt definieren u ( P S ) = S ( Λ 0 ( P ) ) u ( P 0 S ) . Dies ist eine Lösung, die sich bewegende Teilchen mit Spin beschreibt S in seinem Ruherahmen. Wählen wir nun eine beliebige Lorentz-Transformation Λ und berechne seine Wirkung
S ( Λ ) u ( P S ) = S ( Λ Λ 0 ( P ) ) u ( P 0 S ) = S ( Λ 0 ( Λ P ) ) S ( W ( Λ , P ) ) u ( P 0 S ) ,
Wo W ( Λ , P ) = Λ 0 ( Λ P ) 1 Λ Λ 0 ( P ) . Diese Lorentz-Transformation wird Wigner-Rotation genannt. Das ist leicht zu sehen W ( Λ , P ) P 0 = P 0 , also gilt die vorherige Formel. Deshalb
S ( Λ ) u ( P S ) = S ( Λ 0 ( Λ P ) ) S ' D S ' S ( W ( Λ , P ) ) u ( P 0 S ' ) = S ' D S ' S ( W ( Λ , P ) ) u ( Λ P , S ' ) .
Die zweite Gleichheit folgt aus der Definition von u ( P S ) für P P 0 .

Also, in einfachen Worten, Sie sagen, dass die S Komponenten mischen sich unter Lorentz-Transformationen, richtig? Oder anders gesagt, die richtige Formel ist S u S = S ' D S S ' u S ' , anstatt S u S = u S , Rechts?
vielleicht hätte ich schreiben sollen S u S = S ' M S S ' u S ' für irgendeine Matrix M . Ich wollte die allgemeine Struktur der Gleichung schreiben, ohne mich um die Details zu kümmern. Für mich ist der wichtige Teil, dass sie sich mischen, aber nicht, dass die Matrix es ist D oder eine andere Matrix. Wie auch immer, ich werde meinen Beitrag in ein oder zwei Minuten bearbeiten ...
Sie mischen sich, aber nicht durch D ( Λ ) aber eher als durch D ( W ( Λ , P ) ) . Bitte beachten Sie einige wichtige Fakten. Erstens hängt diese Wigner-Rotation explizit vom Impuls des Teilchens ab. Zweitens das D ( Λ ) würde nicht einmal Sinn machen, weil Wigners Matrix D ist für Rotationen und nicht für allgemeine Lorentz-Transformationen definiert. Zweitens können Sie sich mit diesen Formeln davon überzeugen, dass Sie, wenn Sie ein ruhendes Teilchen mit einer bestimmten Spinkonfiguration nehmen, es einmal und dann wieder in eine andere Richtung verstärken, am Ende etwas mit unterschiedlicher Polarisation erhalten.